Đến nội dung

Secrets In Inequalities VP nội dung

Có 298 mục bởi Secrets In Inequalities VP (Tìm giới hạn từ 12-05-2020)



Sắp theo                Sắp xếp  

#381566 Chứng minh tồn tại $k_1,k_2,...,k_s$ để $b_1+m_1k_1=b_2+m_2k_2...

Đã gửi bởi Secrets In Inequalities VP on 29-12-2012 - 14:37 trong Số học

Cho $m_1,m_2,...,m_s$ là các số tự nhiên thỏa $(m_i,m_j)=1$ với mọi $i \ne j$. Cho $b_1,b_2,...,b_s \in \mathbb{Z}$. Chứng minh tồn tại $k_1,k_2,...,k_s \in \mathbb{Z}$ sao cho:
$$b_1+m_1k_1=b_2+m_2k_2=...=b_s+m_sk_s$$

Xét hệ phương trình đồng dư : $x\equiv {b_i} (Mod{m_i})$
Theo định lí thặng dư của Tàu thì hệ luôn có nghiệm $c$.$\Rightarrow c= {b_i}+{m_i}{k_i}$ suy ra $Q.E.D$



#381559 Chứng minh $m\vdots 1979$

Đã gửi bởi Secrets In Inequalities VP on 29-12-2012 - 14:11 trong Số học

Cho m,n là các số nguyên dương sao cho:
$\frac{m}{n}=1-\frac{1}{2}+\frac{1}{3}-\frac{1}{4}+...-\frac{1}{1318}+\frac{1}{1319}$
Chứng minh $m\vdots 1979$

Ta thấy $1979$ là số nguyên tố .
Ta có : $\frac{m}{n}=1-\frac{1}{2}+\frac{1}{3}-\frac{1}{4}+...-\frac{1}{1318}+\frac{1}{1319}$
$$=(1+\frac{1}{2}+\frac{1}{3}+\frac{1}{4}+...+\frac{1}{1318}+\frac{1}{1319})-2.(\frac{1}{2}+\frac{1}{4}+...+\frac{1}{1318})$$
$$=(1+\frac{1}{2}+\frac{1}{3}+\frac{1}{4}+...+\frac{1}{1318}+\frac{1}{1319})-(1+\frac{1}{2}+...+\frac{1}{659})$$
$$=\frac{1}{660}+\frac{1}{661}...+\frac{1}{1318}+\frac{1}{1319}$$
$$=(\frac{1}{660}+\frac{1}{1319})+(\frac{1}{661}+\frac{1}{1338})+...$$
$$=\frac{1979}{660.1319}+\frac{1979}{661.1318}+...$$
$$=1979.(\frac{1}{660.1319}+\frac{1}{661.1318}+...)= 1979.\frac{p}{q}$$
trong đó $q=660.661...1319\Rightarrow (q,1979)=1$
Ta có : $\frac{m}{n}= 1979.\frac{p}{q}\Rightarrow mq=1979np\Rightarrow mq\vdots 1979$
Mà $(q,1979)=1\Rightarrow m\vdots 1979\Rightarrow Q.E.D$



#381558 Với mỗi số nguyên dương n, ký hiệu S(n) là tổng tất cả các chữ số trong biểu...

Đã gửi bởi Secrets In Inequalities VP on 29-12-2012 - 13:55 trong Số học

Với mỗi số nguyên dương n, ký hiệu S(n) là tổng tất cả các chữ số trong biểu diễn thập phân của n.
Xét các số nguyên dương m là bội của 2003. Hãy tìm giá trị nhỏ nhất của S(m).

Đặt $p=2003\in\mathbb{P}$.
+ Nếu $S(n)= 1\Rightarrow n= 100...000= 10^{^{k}}\not\vdots 2003\Rightarrow S(n)> 1$
+ Nếu $S(n)= 2$$\Rightarrow n= 200...000$ hoặc $n=100...100...00= 10^{i}+10^{j}$
Ta có $n=200...000= 2.10^{k} \not\vdots 2003$ $\Rightarrow n=10^i+10^j$
$\Rightarrow 10^i+10^j\vdots 2003=p\Rightarrow 10^i= -10^j(Modp)$
$\Rightarrow 10^{i-j}= -1(Modp)$$\Rightarrow 10^k= -1(Modp)$ ( $k=i-j$ )
Dễ thấy $2^{10}= 1024\equiv 10^7(Modp)$ , suy ra :
$(2^{5k})^{2}= 2^{10k}\equiv (10^{k})^{7}\equiv -1(Modp)$
$\Rightarrow -1$ là số chính phương mod p suy ra $p$ có dạng $4k+1$
$\Rightarrow 2003=p= 4k+1\Rightarrow k= \frac{2002}{4} \not \in \mathbb{Z}$
Suy ra vô lí $\Rightarrow S(n)> 2$
+ Bây giờ ta chứng minh là có $n$ để $S(n)= 3$
Vì $p=2003\neq 8k-1\neq 8k+1$ suy ra $2$ không là SCP mod p $\Rightarrow 2^{\frac{p-1}{2}}\equiv -1(Modp))$
Mà $10^{7}\equiv 2^{10}(Modp)\Rightarrow 2.10^{700}\equiv 2^{1001}= 2^{\frac{p-1}{2}}\equiv -1 (Modp)$
$\Rightarrow (2.10^{700}+1)\vdots p$ .Mà $S(2.10^{700}+1)= 3$ nên khẳng định của ta là đúng.
Vậy $minS(n)= 3$.Xảy ra khi chẳng hạn $n= 2.10^{700}+1$



#369746 $S = C_{2n + 1}^0{.2^{2n}} + C_{2n +...

Đã gửi bởi Secrets In Inequalities VP on 15-11-2012 - 21:56 trong Số học

Đặt $a=1+\sqrt{3},b= 1-\sqrt{3}\Rightarrow ab=-2;\frac{a^2}{2}= 2+\sqrt{3};\frac{b^2}{2}= 2-\sqrt{3}$
${P_n}= \frac{1}{2}(a^{2n+1}+b^{2n+1})$
Dùng khai triển Newton cho $(1+\sqrt{3})^{2n+1},(1-\sqrt{3})^{2n+1}$ta suy ra đc ${P_n}= \sum_{k=0}^{n}\binom{2k}{2n+1}.3^k$ là số nguyên .
Lại dùng Newton cho $(2+\sqrt{3})^{2n+1},(2-\sqrt{3})^{2n+1}$ ta đc:
${S_n}= \frac{(\frac{a^2}{2})^{2n+1}+(\frac{b^2}{2})^{2n+1}}{4}= \frac{a^{4n+2}+b^{4n+2}}{2^{2n+3}}= \frac{a^{4n+2}+2(ab)^{2n+1}+b^{4n+2}}{2^{2n+3}}+\frac{1}{2}= \frac{{P_n}^{2}}{2^{2n+1}}+\frac{1}{2}$
Nhân chéo lên
$\Rightarrow {P_n}^{2}\vdots 2^{2n},{P_n}^{2}\not\vdots 2^{2n+1}\Rightarrow {P_n}^{2}= m2^{2n}$ ( m lẻ )
Mà ${P_n}^{2}$ và $2^{2n}$ chính phuong nên $m= (2p+1)^{2}$
$\Rightarrow {P_n}^{2}= (2p+1)^22^{2n}$
$\Rightarrow {P_n}= (2p+1)2^{^{n}}\Rightarrow\frac{{P_n}-2^n}{2^{n+1}}= p$
Mà dễ thấy :
${S_n}= \frac{{P_n}^{2}}{2^{2n+1}}+\frac{1}{2}= (\frac{{P_n}-2^n}{2^{n+1}})^{2}+(\frac{{P_n}+2^n}{2^{n+1}})^{2}= p^{2}+(p+1)^{2}$
Vậy ta có $Q.E.D$



#385450 \[ a^3+b^3+c^3+6\ge (a+b+c)^2 \]

Đã gửi bởi Secrets In Inequalities VP on 10-01-2013 - 22:27 trong Bất đẳng thức và cực trị

$ a,b,c>0:\; abc=1 $ CMR
\[ a^3+b^3+c^3+6\ge (a+b+c)^2 \]

Lâu lâu ghé box BĐT chém tí :)) !
Theo BĐT $Schur$ ta có :
$$a^{3}+b^{3}+c^{3}+3abc\geq ab(a+b)+bc(b+c)+ca(c+a)$$
$$\Rightarrow 3(a^{3}+b^{3}+c^{3})+9abc\geq 3(ab(a+b)+bc(b+c)+ca(c+a))$$
$$\Rightarrow 4(a^{3}+b^{3}+c^{3})+15abc\geq a^{3}+b^{3}+c^{3}+3(ab(a+b)+bc(b+c)+ca(c+a))+6abc$$
$$\Rightarrow 4(a^{3}+b^{3}+c^{3})+15abc\geq (a+b+c)^3$$
$$\Rightarrow 4(a^{3}+b^{3}+c^{3})+15\geq (a+b+c)^3$$
$$\Rightarrow 4(a^{3}+b^{3}+c^{3})+24\geq (a+b+c)^3+9$$
Theo $AM-GM$ thì :
$$\frac{(a+b+c)^3}{3}+\frac{(a+b+c)^3}{3}+ \frac{(a+b+c)^3}{3}+9\geq 4\sqrt[4]{\frac{(a+b+c)^9}{3}}$$
$$ 4\sqrt[4]{\frac{(a+b+c)^9}{3}}\geq 4\sqrt[4]{\frac{(a+b+c)^8.3\sqrt[3]{abc}}{3}}= 4(a+b+c)^2$$
Do đó : $4(a^{3}+b^{3}+c^{3})+24\geq 4(a+b+c)^2$
Chia $2$ vế cho $4$ ta được ngay $Q.E.D$



#393852 $a_{n}$ là số nguyên dương,$\forall n\in...

Đã gửi bởi Secrets In Inequalities VP on 06-02-2013 - 17:26 trong Dãy số - Giới hạn

Cho dãy số $(a_{n})$:$(a_{n})\left\{\begin{matrix} a_{0}=1\\ a_{n}=(7a_{n-1}+\sqrt{45a_{n-1}^{2}-36})\div 2,\forall n\in \mathbb{N}^{*} \end{matrix}\right.$
Chứng minh:
a)$a_{n}$ là số nguyên dương,$\forall n\in \mathbb{N}$
b)$a_{n+1}a_{n}-1$ là số chính phương,$\forall n\in \mathbb{N}$

a) Bằng qui nạp dễ cm $({a_n})$ là dãy tăng.
Từ hệ thức ban đầu ta suy ra : $$(2a_{n}-7a_{n-1})^{2}= 45a_{n-1}-36$$
Rút gọn ta được : $$2a_{n}^{2}-7a_{n}a_{n-1}+2a_{n-1}^{2}+18=0$$
Thay $n$ bởi $n+1$ :$$2a_{n+1}^{2}-7a_{n+1}a_{n}+2a_{n}^{2}+18=0$$
Trừ từng vế $$\Rightarrow 2a_{n+1}^{2}-2a_{n-1}^{2}+7a_{n}(a_{n-1}-a_{n+1})= 0$$
$$\Rightarrow (a_{n+1}-a_{n-1})(2a_{n+1}-7a_{n}+2a_{n-1})= 0$$
Vì $({a_n})$ là dãy tăng nên $a_{n+1}-a_{n-1}> 0\Rightarrow 2a_{n+1}-7a_{n}+2a_{n-1}=0$
Từ hệ thức này kết hợp qui nạp dễ thấy $Q.E.D$

b) Ta có : $a_{0}=1;a_{1}=5;a_{2}=34;2a_{n+1}-7a_{n}+2a_{n-1}=0$
Qui nạp một tí là ra : $a_{n+1}a_{n-1}-a_{n}^{2}=9$
Thêm bớt thành hằng đẳng thức tí nữa là ra .



#535546 Cho tứ giác ABCD

Đã gửi bởi Secrets In Inequalities VP on 30-11-2014 - 15:34 trong Hình học

Gọi $X,Y$ là giao của $EF$ với $AB$ và $CD$, $K$ là giao của $AB$ và $CD$

Ta có : $(KXAB)=(KYDC)=-1$ $=>KX.KM=KA.KB=KD.KC=KY.KN$ . suy ra tứ giác $XYNM$ nội tiếp

$=>IM.IN=IX.IY=IE^2$ ( Do $(EFYX)=-1$. đpcm




#448240 Chứng minh rằng : $m\vdots p$

Đã gửi bởi Secrets In Inequalities VP on 06-09-2013 - 18:38 trong Số học

CHo $p$ là số nguyên tố lẻ.Kí hiệu : ${S_a}= a+\frac{a^{2}}{2}+...+\frac{a^{p-1}}{p-1}$.

Giả sử ${S_3}+{S_4}-3{S_2}=\frac{m}{n}$. Chứng minh rằng : $m\vdots p$




#414790 $x^2+y^2+z^2-wp=0$

Đã gửi bởi Secrets In Inequalities VP on 25-04-2013 - 20:13 trong Số học

 

CMR: với mọi số nguyên tố cho trước $p$ thì thì tồn tại số tự nhiên $x,y,z,w$ thỏa $x^2+y^2+z^2-wp=0$ và $0<w<p$

 

 

+Nếu $p=2$ chọn ngay $x=0,y=z=w=1$ ta có đpcm

+Nếu $p>2$.CHọn $z=1$.Xét các tập $A={x^2}$, $B={-y^2-1}$ với $x$ và $y$ lấy giá trị trong tập $C$ từ $0$ đến $\frac{p-1}{2}$.

Dễ thấy nếu $a,b\in C$ thì $a^{2}\not\equiv b^{2} (mod p)$ bởi vì nếu ngc lại thì $(a-b)(a+b)\vdots p$ nhg cả hai số này đều nhỏ hơn $p$ nên vô lí.

$\Rightarrow$ các phần tử trong $A$ và $B$ có số dư khác nhau khi chia cho $p$.

Mà $|A |+| B |= p+1> p$ nên tồn tại $2$ phần tử $x,y$ thuộc $A$ và $B$ sao cho $$x^{2}\equiv -y^{2}-1(Modp)\Rightarrow x^2+y^2+1= wp$$

$\Rightarrow 0< w=\frac{x^2+y^2+1}{p}\leq \frac{2(\frac{p-1}{2})^2+1}{p}< p$

Vậy chọn $x,y,z,w$ như trên ta có đpcm.




#414786 $P(x)=(x-a_1)^{2}(x-a_2)^{2}...(x-a_n)^{2}...

Đã gửi bởi Secrets In Inequalities VP on 25-04-2013 - 19:59 trong Đa thức

Chứng minh rằng với mọi số nguyên $a_1,a_2,...,a_n$ đôi một khác nhau,thì đa thức:$P(x)=(x-a_1)^{2}(x-a_2)^{2}...(x-a_n)^{2}+1$ không thể biểu diễn thành tích của hai đa thức (bậc dương) với hệ số nguyên.

Phản chứng giả sử $P(x)=g(x).h(x)$ ,$g(x),h(x) \in R[x]$ , $degg(x)+degh(x)= 2n$,$deg g(x)\leq deg h(x)\Rightarrow degg(x)\leq n$

$\Rightarrow g({a_i}).h({a_i})=P({a_i})= 1$$\forall i=1,2,...,2013$

$\Rightarrow$ $g(x)$ và $h(x)$ cùng đồng nhất bằng $1$ hoặc $-1$.

Nếu có $i,j$ sao cho $g({a_i})= 1; g({a_j})= -1\Rightarrow g({a_i}).g({a_j})< 0\Rightarrow \exists {x_0}:g({x_0})= 0$

$\Rightarrow P(x)$ có nghiệm ${x_0}$ vô lí vì $P(x)$ luôn dương với mọi $x$.

Do đó chỉ có thể xảy ra $g({a_i})=h({a_i})=1$ hoặc $g({a_i})=h({a_i})=-1$. với mọi $i$ chạy từ $1$ đến $n$

Xét TH $g({a_i})=h({a_i})=1$ , cái kia cmtt.

Vì $g({a_i})=1$ nên đa thức $g(x)-1$ có $n$ nghiệm từ ${a_1}$ đến ${a_n}$ nên bậc của nó sẽ lớn hơn hay bằng $n$ 

$degg(x)= degh(x)= n$

$\Rightarrow g(x)-1= c(x-{a_1})(x-{a_2})...(x-{a_n})$ , $ h(x)-1= d(x-{a_1})(x-{a_2})...(x-{a_n})$.

Mà $P(x)= g(x).h(x)$ nên so sánh hệ số cao nhất $2$ vế ta có $cd=1$.Giả sử là $c=d=1$.

Lại do $P(x)= g(x).h(x)$ $\Rightarrow \prod_{i=1}^{n}(x-{a_i})^{2}= (\prod_{i=1}^{n}(x-{a_i})+1).(\prod_{i=1}^{n}(x-{a_i})+1)$

$\Leftrightarrow 2\prod_{i=1}^{n}(x-{a_i})= 0\forall x\in R$.Vô lí.

Vậy giả sử sai và ta có đpcm




#296808 Tính $\widehat{BAC}$

Đã gửi bởi Secrets In Inequalities VP on 27-01-2012 - 14:47 trong Hình học

Để mình chém bài này! Mọi nguoi tu vẽ hình nhá!

Gọi E là trung điểm của AB .Suy ra:$ \widehat{AHE}= \widehat{BAH}$ và $ ME//AC$
Vì $ ME//AC$ nên $\widehat{AME}=\widehat{MAC}$
Mà $\widehat{BAH}=\widehat{MAC}$ nên $\widehat{AHE}=\widehat{AME}$
$ \Rightarrow$ AEHM nội tiếp $ \Rightarrow \widehat{AEM}= \widehat{AHM}= 90^{\circ}\Rightarrow \widehat{BAC}= 90^{\circ}$



#363928 $\sqrt{a+b^{2}}+\sqrt{b+c^{2...

Đã gửi bởi Secrets In Inequalities VP on 22-10-2012 - 20:56 trong Bất đẳng thức và cực trị

Bài toán 2 ( dễ ) Cho $a,b,c> 0$ và $abc=1$. Chứng minh bất đẳng thức sau:

$\frac{3\sum a^{4}b^{4}}{a^{2}+b^{2}+c^{2}}+\frac{8a^{3}}{\left ( bc+a \right )^{3}}+\frac{8b^{3}}{\left ( ca+b \right )^{3}}+\frac{8c^{3}}{\left ( ab+c \right )^{3}}\geq 6$

Công nhận dễ thật ! :P
Theo AM-GM : $3(a^4b^4+b^4c^4+c^4a^4)\geq (a^2b^2+b^2c^2+c^2a^2)^{2}\geq 3a^2b^2c^2(a^2+b^2+c^2)= 3(a^2+b^2+c^2)$
$\Rightarrow \frac{3 (a^{4}b^{4}+b^{4}c^{4}+c^{4}a^{4})}{a^{2}+b^{2}+c^{2}}\geq 3$
Ta cần CM : $\frac{8a^{3}}{\left ( bc+a \right )^{3}}+\frac{8b^{3}}{\left ( ca+b \right )^{3}}+\frac{8c^{3}}{\left ( ab+c \right )^{3}}\geq 3$
$\Leftrightarrow \frac{1}{\left ( 1+\frac{bc}{a} \right )^{3}}+\frac{1}{\left ( 1+\frac{ca}{b} \right )^{3}}+\frac{1}{\left ( 1+\frac{ab}{c} \right )^{3}}\geq \frac{3}{8}$
Đặt $\frac{bc}{a}= x,\frac{ca}{b}= y,\frac{ab}{c}= z\Rightarrow xyz= 1$
$Q.E.D\Leftrightarrow \frac{1}{\left ( 1+x \right )^{3}}+\frac{1}{\left ( 1+y \right )^{3}}+\frac{1}{\left ( 1+z \right )^{3}}\geq \frac{3}{8}$
Quá quen rồi !



#310442 $2\sum a^{4}+4\sum a^{2}b^{2}\geq 3\sum ab(a^{2}+b^{...

Đã gửi bởi Secrets In Inequalities VP on 14-04-2012 - 22:20 trong Bất đẳng thức và cực trị

Một bài vui vuj ! :icon6: !
Cho $a,b,c$ là các số thục ko âm .
CMR : $2(a^{4}+b^{4}+c^{4})+4(a^{2}b^{2}+b^{2}c^{2}+c^{2}a^{2})\geq 3ab(a^{2}+b^{2})+3bc(b^{2}+c^{2})+3ca(c^{2}+a^{2})$.



#308232 $ abc+2\sum a^{2}+8\geq 5\sum a$

Đã gửi bởi Secrets In Inequalities VP on 04-04-2012 - 22:04 trong Bất đẳng thức và cực trị

Cho $ a,b,c\geq 1$ . CMR :
$ abc+2(a^{2}+b^{2}+c^{2})+8\geq 5(a+b+c)$.
-----------------------------------------------------------------------------------
Bài này thục ra ĐK là $ a,b,c\geq 0$ thui , nhg nếu nhu thế thj` sẽ khó hon .
Ai có time thì CM thủ nhé !



#307966 $ \sum \frac{(a+b)^{2}}{a^{2}+b^{2}+2c^{2}}$

Đã gửi bởi Secrets In Inequalities VP on 03-04-2012 - 18:08 trong Bất đẳng thức và cực trị

Cho $ a,b,c> 0$ .CMR:
$ \frac{(a+b)^{2}}{a^{2}+b^{2}+2c^{2}}+\frac{(b+c)^{2}}{b^{2}+c^{2}+2a^{2}}+\frac{(c+a)^{2}}{c^{2}+a^{2}+2b^{2}}$$ \leq 3$



#304410 $$3\left (x^2y + y^2z + z^2x\right )\left (xy^2 + yz...

Đã gửi bởi Secrets In Inequalities VP on 15-03-2012 - 17:15 trong Bất đẳng thức và cực trị

BĐT :
$\Leftrightarrow 3(\frac{x}{z}+\frac{y}{x}+\frac{z}{y})(xy^{2}+yz^{2}+zx^{2})\geq (x+y+z)^{3}$
$\Leftrightarrow (1+1+1)(\frac{x}{z}+\frac{y}{x}+\frac{z}{y})(zx^{2}+xy^{2}+yz^{2})\geq (x+y+z)^{3}$
(luôn đúng theo BĐT Holder)
Dấu "=" xảy ra khi và chỉ khi x = y = z .



#317708 Cho 3 số dương x,y,z thỏa $x+y+z=6$. CMR $x^2 +y^2 +z^2-xy-yz-...

Đã gửi bởi Secrets In Inequalities VP on 18-05-2012 - 21:09 trong Bất đẳng thức và cực trị

Cho 3 số dương x,y,z thỏa $x+y+z=6$. CMR
$x^2 +y^2 +z^2-xy-yz-zx+xyz\geq 8$

Đặt $ x+y+z=p$ , $ xy+yz+zx=q$ , $\ xyz=r$
$ BDT\Leftrightarrow p^{2}-3q+r\geq 8$ $ \Leftrightarrow 36-3q+r\geq 8\Leftrightarrow 28-3q+r\geq 0$
Ta sẽ CM :$28-3q+r\geq 0$ (1)
Tù BĐT quen thuộc : $ (x+y-z)(y+z-x)(z+x-y)\leq xyz$
Ta rút ra đc : $ r\geq \frac{4pq-p^{3}}{9}$ $ \Rightarrow r\geq \frac{8}{3}q+24$
Suy ra : $ VT\geq 28-3q+\frac{8}{3}q-24= 4-\frac{q}{3}$
Ta có BĐT quen thuộc : $ xy+yz+zx\leq \frac{(x+y+z)^{2}}{3}= 12\Rightarrow q\leq 12$
Do đó : $ VT\geq 4-\frac{q}{3}\geq 4-\frac{12}{3}= 0$
Suy ra : (1) đúng $\ \Rightarrow BDT$ đúng.
Dấu"=" xảy ra khi và chỉ khi $ x=y=z=2$ .




#332452 cho a,b,c>0 có tổng bằng 1. CMR $\sum \frac{ab}{\sqrt...

Đã gửi bởi Secrets In Inequalities VP on 06-07-2012 - 10:33 trong Bất đẳng thức và cực trị

Theo BĐT Cauchy-schwarz ta có : $\sum \frac{ab}{\sqrt{ab+bc}}= \sum \frac{a\sqrt{b}}{\sqrt{c+a}}\leq \sum \frac{a\sqrt{2b}}{\sqrt{c}+\sqrt{a}}$
Ta cần CM : $\sum \frac{a\sqrt{2b}}{\sqrt{c}+\sqrt{a}}\leq 1$
Đặt $\sqrt{a}= x,\sqrt{b}= y,\sqrt{c}= z$
BĐT $\Leftrightarrow \sum \frac{2x^{2}y}{z+x}\leq 1= \sum x^{2}$
$\Leftrightarrow \sum x^{2}+\sum (2xy-\frac{2x^{2}y}{z+x})\geq 2\sum xy$$\Leftrightarrow \sum x^{2}+2xyz\sum \frac{1}{z+x}\geq 2\sum xy$
Mà $2xyz\sum \frac{1}{z+x}\geq \frac{9xyz}{x+y+z}$ nên BĐt cần CM đc đua về :
$\sum x^{2}+\frac{9xyz}{x+y+z}\geq 2\sum xy$
$\Leftrightarrow (x^{2}+y^{2}+z^{2})(x+y+z)+9xyz\geq 2(xy+yz+zx)(x+y+z)$
$\Leftrightarrow x^{3}+y^{3}+z^{3}+3xyz\geq ab(a+b)+bc(b+c)+ca(c+a)$
( Đúng theo Schur )
Đ.P.C.M OK



#354131 $t\vdots n$

Đã gửi bởi Secrets In Inequalities VP on 14-09-2012 - 19:48 trong Số học

Cho $n,k$ là các số tự nhiên thỏa mãn n không chia hết cho 3 và $k\geq n$.
Chứng minh rằng luôn tồn tại số nguyên dương t sao cho $t\vdots n$ và $S(t)=k$
$S(t)$ là tổng các chữ số của t



#363919 Chứng minh$(a^2+b+\frac{3}{4})(b^2+a+\frac...

Đã gửi bởi Secrets In Inequalities VP on 22-10-2012 - 20:34 trong Bất đẳng thức và cực trị

Cho các số thực không âm a,b.Chứng minh
$(a^2+b+\frac{3}{4})(b^2+a+\frac{3}{4})\geq (2a+\frac{1}{2})(2b+\frac{1}{2})$.

Theo AM-GM : $VT=[(a^2+\frac{1}{4})+b+\frac{1}{2}][(b^2+\frac{1}{4})+a+\frac{1}{2}]\geq (a+b+\frac{1}{2})^{2}= (a+b)^2+a+b+\frac{1}{4}$
$\geq 4ab+a+b+\frac{1}{4}=(2a+\frac{1}{2})(2b+\frac{1}{2})$



#343716 Chứng minh $(a+b)^2+(a+b+4c)^2\ge \frac{100abc}...

Đã gửi bởi Secrets In Inequalities VP on 05-08-2012 - 18:27 trong Bất đẳng thức và cực trị

(HongKong TST 2001) Cho $a,b,c$ thực dương. Chứng minh $(a+b)^2+(a+b+4c)^2\ge \frac{100abc}{a+b+c}$

BĐT $\Leftrightarrow [(a+b)^2+(a+b+4c)^2](a+b+c)\geq 100abc$
Mà $(a+b)^2+(a+b+4c)^2= (a+b)^2+[(a+b)+4c]^2\geq 4ab+16c(a+b)=4(ab+4bc+4ac)$
Nên ta chỉ cần CM : $2(a+b+c)(ab+4bc+4ca)\geq 50abc$
Đúng vì :
$2(a+b+c)=a+a+b+b+2c\geq 5\sqrt[5]{2a^2b^2c}$

$ab+4bc+4ca= ab+2bc+2bc+2ca+2ca\geq 5\sqrt[5]{16a^3b^3c^4}$
Nhân 2 vế các BĐT trên ta đk ngay Đ.P.C.M .



#343701 Tìm Max của Q = 2(a + b + c) - abc

Đã gửi bởi Secrets In Inequalities VP on 05-08-2012 - 17:35 trong Bất đẳng thức và cực trị

Cho các số a, b, c thỏa mãn $a^2+b^2+c^2=9$. Tìm Max của Q = 2(a + b + c) - abc

Here : http://diendantoanho...ight-leq-10abc/
______________
@BlackSelena : xin phép được Close topic :).



#375185 Tìm tất cả các cấp số cộng có vô hạn số hạng sao cho tồn tại số $N$...

Đã gửi bởi Secrets In Inequalities VP on 04-12-2012 - 21:25 trong Số học

Tìm tất cả các cấp số cộng có vô hạn số hạng sao cho tồn tại số $N$ mà với mọi $p>N$, nếu $a_p$ nguyên tố thì $p$ cũng là số nguyên tố